JEE Main Question Paper with Solution 2022 July 26th Shift 1 - Morning

JEE Main Physics Question Paper with Solution 2022 July 26th Shift 1 - Morning

A

3360 N

B

3380 N

C

3120 N

D

3240 N

Solution

Let acceleration of $100\, kg$ block $= a _1$
FBD of $100\, kg$ block w.r.t ground
image
$F - T - N _1=100 a _1$
FBD of $20$ block wrt $100 \,kg$
image
$ T -20 g =20(2)$
$ T =240 .....$(ii)
$N _1=20 a _1 \ldots \text {.... (iii) }$
FBD of $10\, kg$ block wrt $100\, kg$
image
$ a_1=26 \, m / s ^2 $
$F -240-20(26)=100 \times 26$
$ \Rightarrow F =3360 \,N$

A

4 m

B

20 m

C

30 m

D

50 m

Solution

Given: Frequency $f_1=6\, MHz$
Frequency $f_2=10\, MHz$
$ \lambda_1=\frac{ c }{ f _1}$
$ \lambda_2=\frac{ c }{ f _2}$
Wavelength bandwidth $=\lambda_2-\lambda_1=20 \, m$

A

$0.02 \,\min ^{-1}$

B

$2.7 \,\min ^{-1}$

C

$0.063 \,\min ^{-1}$

D

$6.3 \,\min ^{-1}$

Solution

At $t =0 $ disintegration rate $=4250\, dpm$
At $t =10$ disintegration rate $=2250 \,dpm$
$A=A_o e^{-\lambda t}$
$ 2250=4250 e ^{-\lambda(10)}$
$ \Rightarrow \lambda(10)=\ln \left(\frac{4250}{2250}\right) $
$ \Rightarrow \lambda=0.063 \,\min ^{-1}$

A

$3 \times 10^{16}$

B

$4.5 \times 10^{16}$

C

$4.5 \times 10^{17}$

D

$4.5 \times 10^{20}$

Solution

Wavelength of incident beam $\lambda=900 \times 10^{-9}$
Intensity of incident beam $= I =100 W / m ^2$
No. of photons crossing per unit sec
$ = n =\frac{ E _{\text {net }}}{ E _{\text {single photon }}}=\frac{ IA \lambda}{ hc } $
$ =\frac{(100)\left(1 \times 10^{-4}\right)\left(900 \times 10^{-9}\right)}{6.62 \times 10^{-34} \times 3 \times 10^8}=4.5 \times 10^{16}$

A

16

B

9

C

48

D

12

Solution

For a given light wavelength corresponding a medium of refractive index
$= $_________
and we know that fringe width ____
Therefore, $\beta_{\text {med }}=$ _____ $=\frac{12}{\underline{4}}=9 \,mm$

A

$6\, Vm ^{-1}$ along $x$-axis

B

$3\, Vm ^{-1}$ along z-axis

C

$6\, Vm ^{-1}$ along z-axis

D

$2 \times 10^{-8} Vm ^{-1}$ along z-axis

Solution

$c =\frac{ E _0}{ B _0} \Rightarrow E _0= cB _0 $
$ E _0=\left(3 \times 10^8\right)\left(2 \times 10^{-8}\right) $
$ E _0=6 \,Vm ^{-1}$
As, $\vec{ B }=$ along y-axis $\vec{ v }=$ along negative $x$-axis hence $ \vec{ E }_0=$ along z-axis

A

$\frac{1}{2}$

B

$\frac{1}{\sqrt{2}}$

C

$\frac{\sqrt{3}}{\sqrt{2}}$

D

$2: 1$

Solution

In case of L-R circuit
$Z =\sqrt{ X _{ L }^2+ R ^2} \&$ power factor
$P _1=\cos \phi=\frac{ R }{ Z }$
As $X _{ L }= R$
$\Rightarrow Z =\sqrt{2} R$
$\Rightarrow P _1=\frac{ R }{\sqrt{2} R } \Rightarrow P _1=\frac{1}{\sqrt{2}}$
In case of L-C-R circuit
$Z =\sqrt{ R ^2+\left( X _{ L }- X _{ C }\right)^2}$
As $X_{ L }=X_C$
$ \Rightarrow Z=R $
$ \Rightarrow P_2=\cos \phi=\frac{R}{R}=1$
$ \Rightarrow \frac{P_1}{P_2}=\frac{1}{\sqrt{2}}$

A

3

B

6

C

12

D

2

Solution

As $ \vec{F}=q(\vec{v} \times \vec{B}) $
$ \vec{a}=\frac{q}{m}(\vec{v} \times \vec{B})$
So, $\vec{a} \& \vec{B}$ are $\perp$ to each other
Hence, $\vec{a} \cdot \vec{B}=0$
$ (\alpha \hat{i}-4 \hat{j}) \cdot(2 \hat{i}+3 \hat{j})=0$
$ \alpha(2)+(-4)(3)=0 $
$\alpha=\frac{12}{2} \Rightarrow \alpha=6$

A

$1: 1$

B

$1: 2$

C

$2: 1$

D

$4: 1$

Solution

At centre $B = N \left(\frac{\mu_0 i }{2 R }\right)$
$ B _{ x }=200\left(\frac{\mu_0 i }{2 \times 20 cm }\right) $
$B _{ y }=400\left(\frac{\mu_0 i }{2 \times 20 cm }\right)$
$\frac{ B _{ x }}{ B _{ y }}=\frac{1}{2}$

A

10A

B

20 A

C

4A

D

40A

Solution

image
Given circuit is balanced wheat stone bridge Hence $2 \Omega$ can be neglected
$ R _{ net }=4 \Omega$
$ I =\frac{40}{4}$
$ I=10 A$

A

$200 \,\mu C$

B

$200 \, C$

C

$10 \, \mu C$

D

$10 \, C$

Solution

image
Total charge $= q _1+ q _2+ q _2+ q _4$
$=1 \times 20+2 \times 20+4 \times 20+3 \times 20=200 \,\mu C$

A

Circular

B

Ellipitical

C

Sinusoidal

D

Straight line

Solution

For a particle in SHM, its speed depends on position as
$v =\omega \sqrt{ A ^2- x ^2}$
Where $\omega$ is angular frequency and $A$ is amplitude
Now $v^2=\omega^2 A^2-\omega^2 x^2$
So, $\frac{v^2}{(\omega A )^2}+\frac{x^2}{( A )^2}=1$
So graph between $v$ and $x$ is elliptical

A

$5810 \,J$

B

$3486\, J$

C

$11620\, J$

D

$6972 \,J$

Solution

For a quasi-static process the change in internal energy of an ideal gas is
$\Delta U = nC _{ V } \Delta T $
$ = n \times \frac{3 R }{2} \times \Delta T$
[molar heat capacity at constant volume for mono atomic gas $=\frac{3 R }{2}$ ]
$\Delta U =7 \times \frac{3}{2} \times 8.3 \times 40=3486 \,J$

A

$P$

B

$8 P$

C

$32 P$

D

$64 P$

Solution

Constant entropy means process is adiabatic
$ PV ^\gamma=$ constant
$ V _2=\frac{ V _1}{8} $
$ P _1 V _1^\gamma= P _2 V _2^\gamma $
$P _1 V _1^\gamma= P _2\left(\frac{ V _1}{8}\right)^{5 / 3} $
$ P _1 V _1^{5 / 3}=\frac{ P _2 V _1^{5 / 3}}{32} $
$ P _2=32 P _1$

A

$8.5 \times 10^{-4} J$

B

$8.2 \times 10^{-4} J$

C

$7.5 \times 10^{-4} J$

D

$5.3 \times 10^{-4} J$

Solution

Initial surface energy $= TA$
Where $T$ is surface tension and $A$ is surface area
$ U _{ i }=\left(\frac{75 \times 10^{-5}}{10^{-2}} \frac{ N }{ m }\right) \times\left[4 \pi\left(1 \times 10^{-2}\right)^2\right] $
$=75 \times 10^{-3} \times 4 \pi \times 10^{-4}=942 \times 10^{-7} J$
To get final radius of drops by volume conservation
$\frac{4}{3} \pi R ^3=729\left(\frac{4}{3} \pi r ^3\right)$
$ R =$ Initial radius
$ r =$ final radius
$ r =\frac{ R }{(729)^{1 / 3}}=\frac{ R }{9}=\frac{1}{9} cm$
Final surface energy
$U _{ f }=729[ TA ]$
$=729\left[\frac{75 \times 10^{-5}}{10^{-2}} \frac{ N }{ m }\right] \times\left[4 \pi\left(\frac{1}{9} \times 10^{-2}\right)^2\right] $
$=729\left[75 \times 10^{-3} \times \frac{4 \pi \times 10^{-4}}{81}\right] $
$ =9\left[942 \times 10^{-7} J \right]$
Gain in surface energy $ \Delta U =9 \times 942 \times 10^{-7}-942 \times 10^{-7} $
$ =8 \times 942 \times 10^{-7} J =7536 \times 10^{-7} J$
$ =7.5 \times 10^{-4} J $

A

$1 \%$

B

$3 \%$

C

$4 \%$

D

$0.5 \%$

Solution

Acceleration due to gravity at a height $h << R$ is
$ g ^{\prime}= g \left(1-\frac{2 h }{ R }\right) $
$ \therefore \frac{\Delta g }{ g }=\frac{2 h }{ R } $
$ \Rightarrow \frac{\Delta g }{ g } \times 100=\frac{2 h }{ R } \times 100 $
$=2 \times \frac{32}{6400} \times 100=1 \%$

A

$\frac{ V }{2 \sqrt{2}}$

B

$\frac{v}{2}$

C

$\frac{v}{4}$

D

$\frac{ v }{\sqrt{2}}$

Solution

image
using energy conservation
$\frac{1}{2} mv ^2 \times 2=\frac{1}{2} kx ^2$
$\Rightarrow \frac{1}{4} v ^2=\frac{1}{2} \times 2 \times x ^2$
$\therefore x =\frac{ v }{2}$

A

$T =700 \,N$ while climbing upward

B

$T =350\, N$ while going downward

C

Rope will break while climbing upward

D

Rope will break while going downward

Solution

F.B.D of monkey while moving downward
image
Using Newton's second law
$mg - T = ma _1 $
$\therefore 500- T =50 \times 4 \Rightarrow T =300\, N$
F.B.D of monkey while moving up
image
Using Newton's second law of motion
$T - mg = ma _2 $
$\Rightarrow T -500=50 \times 5$
$\Rightarrow T =750 \,N$
Breaking strength of string $=350 \, N$
$\therefore $ String will break while monkey is moving upward

A

$1: \sqrt{2}$

B

$2: 1$

C

$\sqrt{2}: 1$

D

$1: 2$

Solution

Time taken to reach maximum height
$ t =\frac{ u \sin \theta}{ g } $
$ \therefore \frac{ u _1 \sin \theta_1}{ g }=\frac{ u _2 \sin \theta_2}{ g }$
$ \Rightarrow u _1 \sin 30= u _2 \sin 45$
$ \Rightarrow \frac{ u _1}{ u _2}=\frac{1 / \sqrt{2}}{1 / 2}=\frac{\sqrt{2}}{1}$

A

$6.8 \,cm ^2$

B

$3.4 \,cm ^2$

C

$3.9\, cm ^2$

D

$2.4\, cm ^2$

Solution

L.C. $=\frac{ P }{ N }=\frac{0.5 mm }{50}=0.01\, mm$
Length of wire $=6.8 \, cm$
Diameter of wire $=1.5 \, mm +7 \times$ L.C
$=1.5\, mm +7 \times .01=1.57 \, mm$
Curved surface area $=\pi D \ell$
$=3.14 \times 6.8 \times 1.57 \times 10^{-1} cm ^2$
$=3.352 \, cm ^2=3.4\, cm ^2$

Answer: 5

Solution

$u_x=1 $
$ y=5 x(1-x) $
$ \frac{d y}{d t}=5 \frac{d x}{d t}-10 x \frac{d x}{d t}$
For initial y-component of velocity
$u _{ y }=\left(\frac{ dy }{ dt }\right)_{ x =0} \Rightarrow 5(1)=5 $
$ \vec{ u }_{ y }=5 \hat{ j }$

Answer: 5

Solution

image
using conservation of mechanical energy
$mg 2 R =\frac{1}{2} I _{\text {disc }} \omega^2+\frac{1}{2} I _{\text {particle }} \omega^2$
$mg 2 R =\frac{\omega^2}{2}\left[\frac{ mR ^2}{2}+ mR ^2\right] $
$ mg 2 R =\frac{\omega^2}{2} \frac{3}{2} mR ^2$
$ \frac{3}{4} \omega^2=\frac{2 g }{ R } $
$ \omega^2=\frac{8 g }{3 R } $
$ \omega=\sqrt{\frac{80}{3 R}}$
Given $\omega=4 \sqrt{\frac{x}{3 R}} $
$16 \frac{x}{3 R}=\frac{80}{3 R}$
$x=5$

Answer: 2

Solution

$ L =1 m $
$ \Delta L =0.4 \times 10^{-3} m $
$ m =1 \,kg $
$ d =0.4 \times 10^{-3} m $
$ \frac{ F }{ A }= Y \frac{\Delta L }{ L }$
$ Y =\frac{ FL }{ A \Delta L }=\frac{( mg ) \cdot(1)}{\left(\frac{\pi d ^2}{4}\right) 0.4 \times 10^{-3}} $
$\Rightarrow \frac{10 \times 4}{\pi\left(0.4 \times 10^{-3}\right)^2 \times 0.4 \times 10^{-3}}$
$ Y =\frac{40}{\pi\left(0.4 \times 10^{-3}\right)^3} $
$ Y =\frac{40 \times 7}{22 \times 64 \times 10^{-3} \times 10^{-9}} $
$ Y =0.199 \times 10^{-12} N / m ^2$
$ \frac{\Delta Y }{ Y }=\frac{\Delta F }{ F }+\frac{\Delta L }{ L }+\frac{\Delta A }{ A }+\frac{\Delta(\Delta L )}{(\Delta L )} $
$ =\frac{0.02}{0.4}+2 \frac{\Delta d }{ d }=\frac{0.2}{4}+2 \times \frac{0.01}{0.4}$
$ =\frac{0.1}{2}+\frac{0.1}{2}=0.1$
$ \Rightarrow \Delta Y =0.1 \times Y $
$=0.199 \times 10^{11}=1.99 \times 10^{10}$

Answer: 200

Solution

$ f _1=100= f _0\left(\frac{ C }{ C - V _{ s }}\right) $
$ C =$ speed of sound
$ V _{ s }=$ speed of source
$ f _2=50= f _0\left(\frac{ C }{ C + V _{ s }}\right)$
$ \frac{ f _1}{ f _2}=2=\frac{ C + V _{ s }}{ C - V _{ s }} $
$ 2 C -2 V _{ s }= C + V _{ s } $
$3 V _{ s }= C $
$ V _{ S }=\frac{ C }{3}$
$ 100= f _0 \frac{ C }{\frac{2 C }{3}}=\frac{3}{2} f _0$
$ f _0=\frac{200}{3}$

Answer: 60

Solution

image
Capacitance of each capacitor
$ C _1=\frac{ A 3 \in_0}{\frac{1}{2}}=6 A \in_0 $
$C _2= A 4 \in_0=4 A \in_0$
Equivalent capacitance
$C _{ eq }=\frac{ C _1 C _2}{ C _1+ C _2} \Rightarrow \frac{24}{10} A \in_0 $
$ q _{ net }= C _{ eq }(\Delta V ) \Rightarrow 240 A \in_0$
$\Delta V _2=\frac{240 A \epsilon_0}{4 A \epsilon_0}=60\, V$
$ (\Delta V _2=$ Potential drop across $C _2)$
$V _{\text {foil }}=60\, V$

Answer: 20

Solution

Initially, $\frac{ P }{ Q }=\frac{40\, cm }{60\, cm }=\frac{2}{3} ....$(1)
Finally, $\frac{ P + x }{ Q }=\frac{80 \,cm }{20 \,cm }=\frac{4}{1}.....$(2)
Divide (2) by (1)
$ \frac{P+x}{P}=4 \times \frac{3}{2}=6$
$\Rightarrow 1+\frac{x}{P}=6 \Rightarrow \frac{x}{P}=5$
$ \therefore x=5 P=5 \times 4=20 \Omega$

Answer: 44

Solution

At very high frequencies,
$X _{ C }=\frac{1}{\omega C } \approx 0$
Also $X _{ L }=\omega L \approx \infty$
Thus, equivalent circuit can be redrawn as
image
$ Z =1+2+2=5 \,\Omega $
$I =\frac{220 V }{5 \Omega}=44\, A$

Answer: 10

Solution

image
For point $ B, \frac{1}{ u }=-0.10\, cm ^{-1}, \frac{1}{ v }=0$
$ \therefore$Thus, $u =-10 \,cm , v =\infty $
i.e. $ f =10 \,cm $
$ \Rightarrow \frac{1}{10 cm }=(1.5-1)\left(\frac{2}{ R }\right)=\frac{1}{ R }$
$ \Rightarrow R =10\, cm$

Answer: 5

Solution

For first line of Lyman
$ \frac{1}{\lambda}= R \left(1-\frac{1}{4}\right)= R \left(\frac{3}{4}\right) $
$ \Rightarrow \lambda=\frac{4}{3 R} $
$ 3^{\text {rd }} \text { line(Paschen) } $
$ \frac{1}{\lambda_3}= R \left(\frac{1}{3^2}-\frac{1}{6^2}\right)=\frac{ R }{9} \times \frac{3}{4} $
2nd line(Balmer) $ \frac{1}{\lambda_2}= R \left(\frac{1}{2^2}-\frac{1}{4^2}\right)=\frac{ R }{4} \times \frac{3}{4} $
Thus $ a \lambda=\lambda_3-\lambda_2=\frac{12}{ R }-\frac{16}{3 R }=\frac{20}{3 R }$
putting (1) $ a\left(\frac{4}{3 R}\right)=\frac{20}{3 R} \Rightarrow a=5 $

Answer: 9

Solution

Consider input $120\,V$
image
$I=\frac{(120-60) V }{4000 \Omega}=0.015\, A$
Thus $ I _2= I - I _{ L }$
$=0.015-0.006=0.009\, A =9 \,mA$

JEE Main Chemistry Question Paper with Solution 2022 July 26th Shift 1 - Morning

A

(A) - (I), (B) - (II), (C) - (III), (D) - (IV)

B

(A) - (IV), (B) - (III), (C) - (II), (D) - (I)

C

(A) - (II), (B) - (IV), (C) - (I), (D) - (III)

D

(A) - (III), (B) - (IV), (C) - (II), (D) - (I)

Solution

image
image
image

A

(A) - (III), (B) - (I), (C) - (II), (D) - (IV)

B

(A) - (III), (B) - (II), (C) - (I), (D) - (IV)

C

(A) - (IV), (B) - (III), (C) - (I), (D) - (II)

D

(A) - (IV), (B) - (II), (C) - (III), (D) - (I)

Solution

Sol. $ 2 SO _2( g )+ O _2( g ) \xrightarrow{ V _2 O _5} 2 SO _3( g )$ :
contact process
$4 NH _3( g )+5 O _2( g ) \xrightarrow { Pt ( s )- Rh ( s )} 4 NO ( g )+6 H _2 O ( g ) \text { : }$
Ostwald's process
$N _2( g )+3 H _2( g ) \xrightarrow{ Fe ( s )} 2 NH _3( g ) ; $ Haber's process
Vegetable oil $(l)+ H _2( g ) \xrightarrow{ Ni ( s )}$ vegetable ghee
Hydrogenation

A

Both Statement I and Statement II are correct.

B

Both Statement I and Statement II are incorrect.

C

Statement I is correct but Statement II is incorrect.

D

Statement I is incorrect but Statement II is correct.

Solution

In $Cl _2$ molecule, the covalent radius is half of the internuclear distance, so statement(I) is false.
For the same element, anion has lower effective nuclear charge than atom $\Rightarrow$ so anion is larger than atom. $\Rightarrow$ statement (II) is correct.

A

Low melting point

B

High boiling point

C

High electrical conductivity

D

Less tendency to be soluble in melts than impurities

Solution

Liquation is used to purify metals having lower melting point than impurities present in them.

A

Urea

B

Formaldehyde

C

Formic acid

D

Ethanol

Solution

Urea acts as stabiliser for $H_2O_2$.

A

(A), (D) and (E)

B

(A) , (B) and (D)

C

(D) and (E)

D

(B) , (C) and (D)

Solution

$2 BeCl _2+ LiAlH _4 \rightarrow 2 BeH _2+ LiCl + AlCl _3$

A

$B ( OH )_3$ and $NH _3$

B

$B _2 H _6$ and $NH _3$

C

$B _2 H _6$ and $HN _3$

D

$NH _3$ and $B _2 O _3$

Solution

$3 B _2 H _6+6 NH _3 \xrightarrow{\Delta} 2 B _3 N _3 H _6+12 H _2$

A

$2 H _2 O _2 \rightarrow 2 H _2 O + O _2$

B

$2 NO _2+ H _2 O \rightarrow HNO _3+ HNO _2$

C

$MnO _4^{-}+4 H ^{+}+3 e ^{-} \rightarrow MnO _2+2 H _2 O$

D

$3 MnO _4^{2-}+4 H ^{+} \rightarrow 2 MnO _4^{-}+ MnO _2+2 H _2 O$

A

$C _2 H _6$ and $Cl _2$

B

$\dot{ C } HCl _2$ and $H _2$

C

$\dot{ C } H _3$ and $HCl$

D

$C _2 H _6$ and $HCl$

Solution

$\dot{ Cl } + CH _4 \longrightarrow \dot{ C } H _3+ HCl$

A

Steam distillation

B

2-5 ft long column of silica gel

C

Sublimation

D

Preparative TLC (Thin Layer Chromatography)

Solution

image
Solvent polarity has been related to $R_f$ value of nitrocompounds.
$100\, mg$ p-nitrophenol and picric acid have different $R_f$ value on silica gel plate
$\therefore$ Preparative TLC is best to separate $100 \,mg$ of para nitrophenol and picric acid

A

Hyperconjugation in substrate

B

Polarity of solvent

C

Free radical formation

D

Electromeric effect of the substrate

Solution

image
Difference in reactions is observed due to solvent polarity, which
(i) Ionizes phenol to make more reactive phenoxide ion
(ii) Increases electrophilicity of bromine.

A

B

C

D

Solution

$[10]$ Annulene, although follow $(4 n+2) \pi$ electron rule, but it is non-aromatic due to its non planar nature. It is nonplanar due to repulsion of $C - H$ bonds present inside the ring.

A

(A) > (B) > (C) > (D)

B

(A) > (C) > (D) > (B)

C

(C) > (A) > (D) > (B)

D

(C) > (D) > (B) > (A)

Solution

image
Since diazonium ion is a cation hence it is stabilized by electron donating groups and destabilized by electron withdrawing group. Hence Stability order should be $A > C > D > B$.

A

Cationic detergent

B

Soap

C

Anionic detergent

D

Non-ionic detergent

Answer: 24

Solution

$ ppm =\frac{ W _{ Mg }}{ V _{\text {soln }}} \times 10^6=48 $
$ \Rightarrow W _{ Mg }=\frac{48 \times 2 \times 1000}{10^6} $
$ =48 \times 2 \times 10^{-3} g$
$ n _{ Mg }=\frac{ W _{ Mg }}{24}=\frac{48 \times 2 \times 10^{-3}}{24} $
$ =4 \times 10^{-3}$
Number of $ Mg$ atoms $=4 \times 10^{-3} \times 6.02 \times 10^{23}$
$=4 \times 6.02 \times 10^{20} $
$=24.08 \times 10^{20} $
$ \therefore x =24.08$

Answer: 2

Solution

Let $ W _{ H _2}=40 g \Rightarrow n _{ H _2}=\frac{40}{2}=20$
$W _{ O _2}=60 g \Rightarrow n _{ O _2}=\frac{60}{32}=\frac{15}{8}$
$ P _{ H _2}=\left(\frac{20}{20+\frac{15}{8}}\right) \times 2.2$
$ =\frac{20}{20+1.875} \times 2.2$
$ =\frac{20}{21.875} \times 2.2$
$=2.0114 $
$ \simeq 2.01 \,bar$

Answer: 1758

Solution

$ v_{ e }= x v_{ N } $
$ \lambda_{ e }=\lambda_{ N } $
$ \Rightarrow \frac{ h }{ m _{ e } v_{ e }}=\frac{ h }{ m _{ N } v_{ N }} $
$ v_{ e }=\frac{ m _{ N }}{ m _{ e }} \cdot v _{ N }$
$ =\frac{1.6 \times 10^{-27}}{9.1 \times 10^{-31}} v_{ N }$
$ v_{ e }=1758.24 \times v_{ N } $
$ \therefore x =1758.24$

Answer: 200

Solution

$C ( s )+ O _2( g ) \rightarrow CO _2( g ) ; \Delta H =- x\, kJ / mole$
$ Q = C \Delta T =20 \,kJ \times 2$
$40\, kJ$ heat is released for $2.4 g$ of $C$
For $1$ mole '$C$' :
$Q=\frac{40}{2.4} \times 12$
$=\frac{400}{24} \times 12=200\,kJ / mole$
$Q =\Delta E =\Delta H =200\, kJ \left(\because \Delta n _{ g }=0\right)$
$ x =200$

Answer: 54

Solution

$ n _{ HNO _3}=0.5 \times 0.8$
$ =0.4 $ mole
$\left( n _{ HNO _3}\right)_{\text {remains }}=0.4-\frac{11.5}{63}$
$ =0.4-0.1825 $
$ =0.2175 $
Molarity $=\frac{0.2175}{400} \times 1000$
$ =\frac{0.2175}{0.4} $
$ =0.5437 mole / lit . $
$\simeq 0.54 mole / lit . $
$ =54 \times 10^{-2} mol / lit .$

Answer: 2

Solution

$ K _{ eq }^{\prime}=\frac{1}{\sqrt{ K _{ eq }}}=\frac{1}{\sqrt{2 \times 10^{15}}}= x \times 10^{-8} $
$\Rightarrow \frac{1}{\sqrt{20}} \times \frac{1}{10^7}= x \times 10^{-8} $
$\Rightarrow \frac{1}{\sqrt{20}} \times 10^{-7}= x \times 10^{-8} $
$\frac{10}{\sqrt{20}}= x $
$ \Rightarrow x =\frac{\sqrt{10}}{\sqrt{2}}=\sqrt{5}=2.236 $
$ \simeq 2.24$

Answer: 3

Solution

$Fe _3 O _4 \xrightarrow{+8 e ^{-}} 3 Fe$
Charge for 1 mole $Fe =8 / 3 F$
$=2.67 F$

Answer: 2

Solution

$ t _{\frac{1}{2}} \propto \frac{1}{\left[ A _0\right]^{ n -1}} $
$ {[100] \propto \frac{1}{(0.5)^{ n -1}}}$
$ (50) \propto \frac{1}{(1)^{n-1}} $
${[2]^1=\left[\frac{1}{0.5}\right]^{n-1}}$
$ {[2]^1=[2]^{n-1}} $
$ n-1=1 $
$ n=2$ order $=2$

JEE Main Mathematics Question Paper with Solution 2022 July 26th Shift 1 - Morning

A

4

B

10

C

11

D

16

Solution

$ f(x)-f(x / 3)=x / 3$
$ f(x / 3)-f\left(x / 3^2\right)=x / 3^2$
.... on adding
$f(x)-\displaystyle\lim _{n \rightarrow \infty} f\left(\frac{x}{3^n}\right)=x\left(\frac{1}{3}+\frac{1}{3^2} \ldots \infty\right) $
$ f(x)-f(0)=\frac{x}{2}$
$ f(8)=7 ; f(0)=3 $
$ f(x)=x / 2+3 $
$ f(14)=10$

A

$\arg z _2=\pi-\tan ^{-1} 3$

B

$\arg \left( z _1-2 z _2\right)=-\tan ^{-1} \frac{4}{3}$

C

$\left|z_2\right|=\sqrt{10}$

D

$\left|2 z_1-z_2\right|=5$

Solution

$AB = AO \cdot z ^{-i \pi / 2}=-2+i$
So $OB =(-2+i)+(1+2 i)$
$ z _2=-1+3 i $
$ \therefore\left|2 z _1- z _2\right|=\sqrt{10}$

A

$3 \sqrt{5}$

B

4

C

$\frac{26}{9}$

D

$\frac{10}{3}$

Solution

$D = \begin{vmatrix} 8 & 1 & 4 \\1 & 1 & 1 \\\lambda & -3 & 0\end{vmatrix}=0 \Rightarrow \lambda=4$
Also $D _1= D _2= D _3=0$
So $\mu=-2$
Point $\left(4,-2,-\frac{1}{2}\right)$
Distance from plane $=\frac{10}{3}$

A

$-1$

B

2

C

1

D

$-\sqrt{2}$

Solution

Let $A=\begin{bmatrix} a & b \\ c & d \end{bmatrix} ; ad - bc =-1$
$ | A + I ||\operatorname{adj} A + I |=4$
$ \Rightarrow ad - bc + a + d +1=2 \text { or }-2$
$ a + d =2 \text { or }-2$

A

3

B

5

C

7

D

9

Solution

$ A =\int\limits_1^3 y ^{ a } \cdot dy =\left.\frac{ y ^{ a +1}}{ a +1}\right|_1 ^3=\frac{364}{3} $
$ \Rightarrow a =5$

A

560

B

1540

C

1330

D

2600

Solution

$ \left(\left(2^1 2^2 \ldots .2^{60}\right)\left(4^1 \cdot 4^2 \ldots \ldots 4^{ n }\right)\right)^{\frac{1}{60+ n }}=2^{\frac{225}{8}} $
$ \left(2^{30 \times 61} 4^{\frac{ n ( n +1)}{2}}\right) \frac{1}{60+ n }=2^{\frac{225}{8}} $
$ 2^{1830+ n ^2+ n }=2^{\frac{(225)(60+ n )}{8}} $
$ =8 n ^2-217 n +1140=0 $
$ n =20, \frac{57}{8} $
$\displaystyle \sum_{ k =1}^{ n } nk - k ^2=\frac{ n ^2( n +1)}{2}-\frac{ n ( n +1)(2 n +1)}{6} $
$=1330$

A

1

B

-1

C

e

D

0

Solution

$ \displaystyle\lim _{x \rightarrow 0} \frac{\left(\ln \left(1+x^2+x^4\right)\right) \cos x}{1-\cos ^2 x} $
$ \displaystyle\lim _{x \rightarrow 0} \frac{\left(\frac{\ln \left(1+x^2+x^4\right)}{x^2+x^4}\right) x^2\left(1+x^2\right) \cos x}{\left(\frac{\sin ^2 x}{x^2}\right) x^2}=1 $
$ \therefore k=1$

A

-10

B

10

C

8

D

-8

Solution

$f(x)= \begin{cases}x+a ; x \leq 0 \\|x-4| ; x>0\end{cases} ; g(x)= \begin{cases} x+1 ; x<0 \\(x-4)^2+b ; x \geq 0\end{cases}$
For continuity $a=4$ and $b=-15$
$g ( f (2))+ f ( g (-2)) $
$ = g (2)+ f (-1)=-8$

A

$(-6,-2)$

B

$(2,6)$

C

$[-6,-2) \cup(2,6]$

D

$[-\sqrt{6},-2) \cup(2, \sqrt{6}]$

Solution

$ f (1)=3 $
For $ x < 1, f ^{\prime}( x )=3 x ^2-2 x +10>0$
$ \Rightarrow f ( x ) $ is increasing
For $ x >1, f ^{\prime}( x ) < 0 $
$ \Rightarrow $ function is decreasing.
$\displaystyle\lim _{ x \rightarrow 1^{+}} f ( x )=-2+\log _2\left( b ^2-4\right) $
For maximum value at $ x =1$
$ 3 \geq-2+\log _2\left( b ^2-4\right) $
$ 32 \geq b ^2-4>0$
$ b \in[-6,-2) \cup(2,6]$

A

$2 \sqrt{2} f \left(\frac{ a }{2}\right)= f ^{\prime}\left(\frac{ a }{2}\right)$

B

$f\left(\frac{a}{2}\right) f^{\prime}\left(\frac{a}{2}\right)=\sqrt{2}$

C

$\sqrt{2} f \left(\frac{ a }{2}\right)= f ^{\prime}\left(\frac{ a }{2}\right)$

D

$f \left(\frac{ a }{2}\right)=\sqrt{2} f ^{\prime}\left(\frac{ a }{2}\right)$

Solution

$ a =\frac{1}{ n } \displaystyle\sum_{ k =1}^{ n } \frac{2}{1+\left(\frac{ k }{ n }\right)^2}=\int_0^1 \frac{2}{1+ x ^2} dx =\frac{\pi}{2} $
$ f ( x )=\tan \left(\frac{ x }{2}\right) ; x \in(0,1) $
$ f \left(\frac{\pi}{4}\right)=\sqrt{2}-1 $
$ f ^{\prime}\left(\frac{\pi}{4}\right)=\frac{1}{2} \sec ^2\left(\frac{\pi}{8}\right)=\frac{\sqrt{2}}{\sqrt{2}+1}$
$ f ^{\prime}\left(\frac{\pi}{4}\right)=\sqrt{2} f \left(\frac{\pi}{4}\right)$

A

$\frac{9}{2}$

B

$\frac{3 \sqrt{17}}{2}$

C

$\frac{3 \sqrt{17}}{4}$

D

$9$

Solution

$ ( x -1)^2+( y -2)^2+( x +2)^2+( y -1)^2=14$
$\Rightarrow x ^2+ y ^2+ x -3 y -2=0$
Put $ x =0$
$ \Rightarrow y ^2-3 y -2=0$
$ \Rightarrow y =\frac{3 \pm \sqrt{17}}{2}$
Put $ y =0 $
$ \Rightarrow x ^2+ x -2=0 $
$ ( x +2)( x -1)=0$
$\therefore A (-2,0), B (1,0), C \left(0, \frac{3+\sqrt{17}}{2}\right), D \left(0, \frac{3-\sqrt{17}}{2}\right) $
Area $=\frac{1}{2} \cdot 3 \cdot \sqrt{17}=\frac{3 \sqrt{17}}{2}$

A

$(25,10)$

B

$(20,12)$

C

$(30,8)$

D

$(15,13)$

Solution

Tangent at $(\alpha, \beta)$ has slope 1
$\beta^2=24 \alpha$
Equation of tangent $y \beta=12( x +\alpha), \frac{12}{\beta}=1$
$ \Rightarrow \alpha=6, \beta=12$
$ \therefore(\alpha+4, \beta+4)=(10,16)$
Normal at $(10,16)$ to $\frac{x^2}{36}-\frac{y^2}{144}=1$ is
$2 x+5 y=100$

A

$\sqrt{\frac{21}{2}}$

B

$\sqrt{\frac{9}{2}}$

C

$\sqrt{\frac{73}{2}}$

D

1

Solution

d.r's of the line $= \begin{vmatrix} \hat{i} & \hat{j} & \hat{k} \\ 1 & 1 & -1 \\ 1 & -2 & 3\end{vmatrix} =\hat{i}-4 \hat{j}-3 \hat{k}$
$\therefore$ equation of line is
$\vec{ r }=\hat{ i }+2 \hat{ j }+4 \hat{ k }+\lambda(\hat{ i }-4 \hat{ j }-3 \hat{ k })$
Let $A (1,2,4)$ and $Pb (1+\lambda, 2-4 \lambda, 4-3 \lambda)$
$ \therefore \overrightarrow{ PA } \cdot(\hat{ i }-4 \hat{ j }-3 \hat{ k })=0 $
$ \lambda=\frac{1}{2} $
$\Rightarrow P \left(\frac{1}{2}, 2, \frac{-5}{2}\right)$
$ | AP |=\sqrt{\frac{21}{2}}$

A

$\frac{15}{2}$

B

8

C

$\frac{13}{2}$

D

7

Solution

$ \vec{a} \times \vec{b}=(1-\alpha) \hat{i}+\left(\alpha^2-2\right) \hat{j}+(\alpha-2) \hat{k}$
Projection of $ \vec{a} \times \vec{b} \text { on }-\hat{i}+2 \hat{j}-2 \hat{k} $
$ =\frac{(\vec{a} \times \vec{b}) \cdot(-\hat{i}+2 \hat{j}-2 \hat{k})}{3}=30$
$ \Rightarrow 2 \alpha^2-\alpha-91=0$
$ \Rightarrow \alpha=7,-\frac{13}{2}$

A

$\frac{5}{9}$

B

$\frac{64}{81}$

C

$\frac{16}{27}$

D

$\frac{145}{243}$

Solution

$ n p =\alpha ....$ (1)
$ npq =\alpha / 3.....$(2)
From (1) & (2)
$q =1 / 3 \& p =2 / 3 $
$ { }^{ n } C _1 q ^{ n -1} p ^1=\frac{4}{243}$
$ \frac{ n }{3^{ n }}=\frac{2}{243}$
$ n =6$
$P (4 \text { or } 5)$
$={ }^6 C _4\left(\frac{2}{3}\right)^4\left(\frac{1}{3}\right)^2+{ }^6 C _5\left(\frac{2}{3}\right)^5 \cdot\left(\frac{1}{3}\right)^0 $
$ =\frac{16}{27}$

A

$\frac{2}{3}$

B

$\frac{5}{3}$

C

$\frac{5}{4}$

D

1

Solution

$ 0 \leq P \left( E _i\right) \leq 1 \text { for } i =1,2,3 $
$ \Rightarrow-2 / 3 \leq p \leq 1 $
$ E _1 \& E _2 \& E _3 $ are mutually exclusive
$ P \left( E _1\right)+ P \left( E _2\right)+ P \left( E _3\right) \leq 1$
$\Rightarrow 2 / 3 \leq p \leq 1 $
$p _1=1, p _2=2 / 3 $
$ p _1+ p _2=5 / 3$

A

0

B

$-2$

C

$-4$

D

12

Solution

$ 8^{2 \sin ^2 \theta}+8^{2-2 \sin ^2 \theta}=16 $
$ y +\frac{64}{ y }=16$
$ \Rightarrow y =8$
$ \Rightarrow \sin ^2 \theta=1 / 2$
$ n ( S )+\displaystyle\sum_{\theta \in S } \frac{1}{\cos (\pi / 4+2 \theta) \sin (\pi / 4+2 \theta)}$
$ =4+(-2) \times 4=-4$

A

1

B

2

C

$\frac{1}{4}$

D

$\frac{5}{4}$

Solution

$\tan \left(2\left(\tan ^{-1} \frac{1}{5}+\tan ^{-1} \frac{1}{8}\right)+\tan ^{-1}\left(\frac{1}{2}\right)\right) $
$ =\tan \left[2 \tan ^{-1}\left(\frac{1}{3}\right)+\tan ^{-1}\left(\frac{1}{2}\right)\right] $
$ =2$

A

a tautology

B

a contradiction

C

equivalent to $(p \Rightarrow q) \wedge q$

D

equivalent to $( p \Rightarrow q ) \wedge p$

Answer: 180

Solution

$3 \times \frac{5 !}{2 ! 2 !}+\frac{5 !}{3 ! \times 2 !}+\frac{5 !}{2 !}+\frac{5 !}{3 !}=180$

Answer: 3

Solution

$ x^5\left(x^3-x^2-x+1\right)+x\left(3 x^3-4 x^2-2 x+4\right)-1 =0$
$\Rightarrow(x-1)^2(x+1)\left(x^5+3 x-1\right)=0$
Let $ f(x)=x^5+3 x-1 $
$ f^{\prime}(x)>0 \forall x \in R$
Hence $3$ real distinct roots.

Answer: 23

Solution

Since coefficient of $x$ is $-3$
$ \Rightarrow{ }^{ p } C _1-{ }^{ q } C _1=-3$
$\Rightarrow p - q =-3 .....$(1)
Comparing coefficients of $x ^2$
$ -{ }^{ p } C _1{ }^{ q } C _1+{ }^{ p } C _2+{ }^{ q } C _2=-5 .......$(2)
$ - pq +\frac{ p ( p -1)}{2}+\frac{ q ( q -1)}{2}=-5$
Solving (1) and (2)
$p =8, q =11$
Coefficient of $x ^3$ is
$ -{ }^{ q } C _3+{ }^{ p } C _3+{ }^{ p } C _1{ }^{ q } C _2-{ }^{ p } C _2{ }^{ q } C _1 $
$=-{ }^{11} C _3+{ }^8 C _3+{ }^8 C _1{ }^{11} C _2-{ }^8 C _2{ }^{11} C _1 $
$=23$

Answer: 24

Solution

Let $I _1=\int\limits_0^1\left(1- x ^{ n }\right)^{2 n } dx$,
$I_2=\int\limits_0^1\left(1-x^n\right)^{2 n+1} d x$
$I_2=\int\limits_0^1\left(1-x^n\right)^{2 n+1} \cdot 1 d x$
$=\left.\left(1-x^n\right)^{2 n+1} \cdot x\right|_0 ^1-\int\limits_0^1(2 n+1)\left(1-x^n\right)^{2 n}\left(-n x^{n-1}\right) x d x$
$I_2=-n(2 n+1)\left\{I_2-I_1\right\} $
$\left(2 n^2+n+1\right) I_2=n(2 n+1) I_1 $
$\frac{I_1}{I_2}=\frac{2 n^2+ n +1}{ n (2 n +1)}=\frac{1177}{ n (2 n +1)} $
$\Rightarrow 2 n ^2+ n -1176=0 \Rightarrow n =24$

Answer: 6

Solution

$ x^4=3 y x \cdot y^{\prime}-3 y^2$
$ \Rightarrow 3 x y \frac{d y}{d x}=3 y^2+x^4 $
Put $ y^2=t, y \frac{d y}{d x}=\frac{1}{2} \frac{d t}{d x} $
$ \frac{d t}{d x}-\frac{2}{x} t=\frac{2}{3} x^3$
$ \therefore \frac{t}{x^2}=\frac{x^2}{3}+C $
$ \Rightarrow \frac{y^2}{x^2}=\frac{x^2}{3}-2$
Put $(3,3), C=-2$
$ \therefore \frac{y^2}{x^2}=\frac{x^2}{3}-2$
$3 y^2=x^4-6 x^2 $
$ x^4-6 x^2=1080$
$ \therefore x=6$

Answer: 3

Solution

Coordinates of $A (1,-2), B \left(\frac{15 a }{1-2 p }, \frac{-30 a }{1-2 p }\right)$ and orthocentre $H (2$, a)
Slope of $AH = p$
$a+2=p .....$(1)
Slope of $BH =-1$
$31 a -2 ab =15 a +4 p -2 .....$(2)
From (1) and (2)
$a =1 \& p =3$

Answer: 45

Solution

image
$ f^{\prime}(x)=4 x-\frac{1}{x} $
$ a=\frac{1}{2}$
Let $P \left( x _1, y _1\right)$ be any point on $y ^2=4 ax$
$\frac{1}{y_1}=\frac{3-y_1}{4-x_1} \Rightarrow y_1^2-6 y_1+8=0$
$ y_1=2,4 $
$ \Rightarrow P(8,4) $ as $P(2,2) $ rejected
Equation of normal at $ P$ .
$y-4=-4(x-8) $
$ \frac{x}{9}+\frac{y}{36}=1 $
$ \alpha=9, \beta=36 $
$\alpha+\beta=45$

Answer: 153

Solution

Let $(2 \lambda-1,3 \lambda-2,2 \lambda+1)$ be any point on the line
$ (2 \lambda-5)^2+(3 \lambda-4)^2+(2 \lambda-6)^2=26 $
$ \lambda=1,3 $
$ Q (1,1,3) ; R (5,7,7) ; P (4,2,7) $
Area of triangle $ PQR =1 / 2|\overrightarrow{ PQ } \times \overrightarrow{ PR }|$
$ =\sqrt{153}$